logo-home

Charlene's Scholastic Emporium.

Your Actual and Virtual Exam Tests Excellent Tutor.

Tutor EmilyCharlene helps you study by writing summaries of study material

EmilyCharlene helps with

Show more

Community

  • Followers
  • Following

17 Reviews received

13 items

PSI FNP Practice Questions and Answers 100% Pass

(0)
$13.49
0x  sold

PSI FNP Practice Questions and Answers 100% Pass A 14yo male with bronchitis is being treated with fluids and expectorants. He returns to the clinic with a fever of 103F, right pleuritic chest pain, and green sputum. Which of the following examination results would be expected? A. Right lower lobe crackles B. Decreased Fremitus C. Bilateral Wheezing D. Normal Percussion - Correct Answer ️️ -A. Right lower lobe crackles. 2 year old child is diagnosed with radial head subluxation ("nu...

i x
  •  Package deal
  • Exam (elaborations)
  •  • 46 pages • 
  • by EmilyCharlene • 
  • uploaded  23-04-2024
Quick View
i x

LEIK FNP GENDER HEALTH EXAM QUESTIONS AND ANSWERS 100% PASS

(0)
$13.49
0x  sold

LEIK FNP GENDER HEALTH EXAM QUESTIONS AND ANSWERS 100% PASS Which of the following findings is most likely in a young primigravida with pregnancy- induced hypertension? A) Abdominal cramping and constipation B) Edema of the face and the upper extremities C) Shortness of breath D) Dysuria and frequency - Correct Answer ️️ -B) Edema of the face and the upper extremities Common signs and symptoms of pregnancy-induced hypertension include edema of the face and the upper extremities, we...

i x
  •  Package deal
  • Exam (elaborations)
  •  • 61 pages • 
  • by EmilyCharlene • 
  • uploaded  23-04-2024
Quick View
i x

ANCC FNP Exam Review Questions LEIK part 1 Practice Questions and Answers 100% Pass

(0)
$12.49
0x  sold

ANCC FNP Exam Review Questions LEIK part 1 Practice Questions and Answers 100% Pass The Lachman Maneuver is used to detect which of the following? A: Instability of the knee B: Nerve Damage of the knee due to past knee injuries C: Integrity of the patellar tendon D: Tears of the meniscus - Correct Answer ️️ -instability of the knee A 28-year old multipara who is at 32 weeks gestation presents to your office complaining of a sudden onset of small amounts of bright-red vaginal bleedin...

i x
  •  Package deal
  • Exam (elaborations)
  •  • 14 pages • 
  • by EmilyCharlene • 
  • uploaded  23-04-2024
Quick View
i x